Encore un calcul d'intégrale (double)

Il y a 3 ans je me demandais comment évaluer (d'une autre façon) l'intégrale suivante:

$\displaystyle J=\int_0^1 \frac{\ln^2 x}{1+x^2}\,dx$


En considérant l'intégrale suivante:

$\begin{align}K=\int_0^\infty \int_0^\infty \frac{\ln^2 (xy)}{(1+x^2)(1+y^2)}\,dx\,dy\end{align}$

(les bornes infinies sont essentielles ici, ce qui m'avait échappé dans le passé)

Pourquoi cette dernière intégrale?

$\begin{align}K&=\int_0^\infty \int_0^\infty \frac{\ln^2 x+2\ln x\ln y+\ln^2 y}{(1+x^2)(1+y^2)}\,dx\,dy\\
&=\int_0^\infty \int_0^\infty \frac{\ln^2 x}{(1+x^2)(1+y^2)}\,dx\,dy+2\int_0^\infty \int_0^\infty \frac{\ln x\ln y}{(1+x^2)(1+y^2)}\,dx\,dy+\int_0^\infty \int_0^\infty \frac{\ln^2 y}{(1+x^2)(1+y^2)}\,dx\,dy\\
&=2\int_0^\infty \frac{1}{1+x^2}\,dx\int_0^\infty\dfrac{\ln^2 y}{1+y^2}\,dy+\left(\int_0^\infty \frac{\ln x}{1+x^2}\,dx\right)^2\\
&=2\int_0^\infty \frac{1}{1+x^2}\,dx\int_0^\infty\dfrac{\ln^2 y}{1+y^2}\,dy
\end{align}$

puisque $\displaystyle \int_0^\infty \frac{\ln x}{1+x^2}\,dx=0$

Dans l'intégrale $\displaystyle \int_0^\infty\dfrac{\ln^2 y}{1+y^2}\,dy$ on fait le changement de variable $u=\dfrac{1}{y}$,

$\begin{align}\int_0^\infty\dfrac{\ln^2 y}{1+y^2}\,dy&=\int_0^1\dfrac{\ln^2 y}{1+y^2}\,dy+\int_1^\infty\dfrac{\ln^2 y}{1+y^2}\,dy\\
\end{align}$

Dans la deuxième intégrale on fait le changement de variable $u=\dfrac{1}{y}$,

$\begin{align}\int_0^\infty\dfrac{\ln^2 y}{1+y^2}\,dy&=\int_0^1\dfrac{\ln^2 y}{1+y^2}\,dy+\int_0^1\dfrac{\ln^2 u}{1+u^2}\,du\\
&=2J
\end{align}$


Ainsi,

$\begin{align}K&=2\pi J\end{align}$

Dans l'intégrale qui définit $K$ on fait le changement de variable $u=xy$ (on considère la variable $x$ ici),

$\begin{align}K&=\int_0^\infty \int_0^\infty \frac{y\ln^2 u}{(u^2+y^2)(1+y^2)}\,du\,dy\\
&=\int_0^\infty \int_0^\infty \frac{\ln^2 u}{u^2-1}\left(\frac{y}{1+y^2}-\frac{y}{u^2+y^2}\right)\,du\,dy\\
&=\frac{1}{2}\int_0^\infty \frac{\ln^2 u}{u^2-1}\left[\ln\left(\frac{1+y^2}{u^2+y^2}\right)\right]_{y=0}^{y=\infty} \,du\\
&=\int_0^\infty \frac{\ln^3 u}{u^2-1}\,du\\
&=\int_0^1 \frac{\ln^3 u}{u^2-1}\,du+\int_1^\infty \frac{\ln^3 u}{u^2-1}\,du
\end{align}$

Dans la dernière intégrale on fait le changement de variable $x=\dfrac{1}{u}$,

$\begin{align}K&=2\int_0^1 \frac{\ln^3 x}{x^2-1}\,dx\\
&=2\int_0^1 \frac{\ln^3 x}{x-1}\,dx-2\int_0^1 \frac{x\ln^3 x}{x^2-1}\,dx\\
\end{align}$

Dans la dernière intégrale on fait le changement de variable $u=x^2$,

$\begin{align}K&=\frac{15}{8}\int_0^1 \frac{\ln^3 x}{x-1}\,dx\\
\end{align}$

En développant en série entière $\dfrac{1}{1-x}$ on peut montrer que,

$\begin{align}K&=\frac{15}{8}\times 6\sum_{n=0}^{\infty} \frac{1}{(n+1)^4}\\
&=\frac{45}{4}\zeta(4)
\end{align}$

Ainsi,

$\boxed{ \displaystyle J=\frac{45\zeta(4)}{8\pi}}$


PS:
$\zeta(4)=\frac{\pi^4}{90}$ mais si on sort cette formule du chapeau pour obtenir $J=\frac{\pi^3}{16}$ on peut aussi bien sortir directement celle-ci:
$\displaystyle \int_0^1 \frac{\ln^2 x}{1+x^2}\,dx=\frac{\pi^3}{16}$.

Réponses

  • Je sais que FdP n'aime pas l'analyse complexe, mais celle-là, elle se fait VRAIMENT facilement par résidus. :-D

    Par le changement de variable $x=1/u$ on trouve $$J = \int_{1}^{+\infty} \frac{\ln^2 u}{u^2+1} du.$$ Dès lors $$I = \frac{1}{2} \int_{0}^{+\infty} \frac{\ln^2 x}{1+x^2} dx.$$ Soit $\ln_{\pi}$ la fonction logarithme complexe avec coupure sur $[0,+\infty[$. En considérant un contour pacman, il "suffit" de calculer $$\int_{C} \frac{\ln_{\pi}^3(z)}{1+z^2} dz.$$ Peut-être YvesM voudra-t-il le faire. :-D
  • Le passage par $K$ me semble étrange. Perso, je poserais $x=e^{-u}$, et après il me semble que ça se calcule bien en développant en série.
  • Aléa:

    Je suis curieux de voir ça ;)

    $\displaystyle J=2\sum_{n=0}^{\infty} \frac{(-1)^n}{(2n+1)^3}=\dfrac{\pi^3}{16}$
  • Je pense qu'on peut généraliser.

    Soit pour $n>0$ entier,

    $\displaystyle J_n=\int_0^{\infty} \frac{\ln^{2n} x}{1+x^2}\,dx$
    $\displaystyle K_n=\int_0^\infty \int_0^\infty\frac{\ln^{2n} (xy)}{(1+x^2)(1+y^2)}\,dx\,dy$
    A noter que pour tout $n\geq 0$ entier $\displaystyle \int_0^{\infty} \frac{\ln^{2n+1} x}{1+x^2}\,dx=0$

    Calculons $J_2$.


    $\begin{align} K_2&=\int_0^\infty \int_0^\infty\frac{\ln^4 (xy)}{(1+x^2)(1+y^2)}\,dx\,dy\\
    &=\pi J_2+6J_1^2\\
    \end{align}$

    D'autre part, en faisant le même calcul que précédemment on peut montrer que,

    $\begin{align}K_2&=\int_0^\infty\frac{\ln^5 (x)}{x^2-1}\,dx\end{align}$

    Et par la même technique que précédemment on peut calculer cette dernière intégrale en fonction de $\zeta(4)$,

    $\begin{align}K_2&=\frac{945}{4}\zeta(6)\end{align}$

    Donc,

    $\begin{align}K_2&=\frac{945}{4}\zeta(6)\end{align}$

    On peut donc calculer $J_2$ en fonction de $\zeta(6),\pi$ et de $J_1$ qu'on a précédemment calculé.

    On devine qu'on peut calculer de proche en proche les valeurs de $J_n$ en fonction de valeurs de $\zeta$ et en fonction de $\pi$.
  • Pourquoi tant d'energie ? la formule des complements suffit. Pour $|s|<1/2$
    $$\int_0^{\infty}\frac{y^{s+\frac{1}{2}-1}}{1+y}=B(\frac{1}{2}+s,\frac{1}{2}-s)=\frac{\pi}{\mathrm{cos}\ \pi s}.$$ et hop, avec $r_n$ un rationnel de moins en moins facile a calculer :
    $$\int_0^{infty}\frac{(\log y)^{2n}}{(1+y)\sqrt{y}}dy=\left(\frac{d}{ds}\right)^{2n}\frac{\pi}{\mathrm{cos}\ \pi s}\big|_{s=0}=r_n\pi^{2n+1},\ \ \ \ \frac{1}{\cos x}=\sum_{n=0}^{\infty}\frac{r_n}{(2n)!}x^{2n}.$$
  • Quand la formule des compléments est là, P. n'est jamais très loin ... Bravo.

    Sinon, pour la petite histoire, les $r_n$ sont connus sous le nom de nombres d'Euler.
  • P.:

    La formule que tu utilises n'est pas élémentaire.


    PS:

    Je proposais une méthode de calcul effective de ces constantes ($J_n,K_n$)
  • Tout ça, ce sont des questions de goûts personnels.
    C'est assez dur d'avoir des preuves qui sont à la fois naturelles et peu coûteuses en matériel.
    Je sais bien qu'il y a en France (et en particulier en taupe) une certaine tradition de présenter des preuves parachutées de résultats avancés avec des outils élémentaires. Je ne suis pas sûr qu'on y gagne.

    La formule des compléments se démontre de manière assez naturelle avec des calculs des résidus, ou de manière moins naturelle (de mon point de vue), mais élémentaire, avec les séries de Fourier.

    Si on connait la formule de Faa di Bruno, la méthode de P. est aussi "effective".
  • Alea a écrit:
    La formule des compléments se démontre de manière assez naturelle avec des calculs des résidus, ou de manière moins naturelle (de mon point de vue), mais élémentaire, avec les séries de Fourier.

    Des calculs non élémentaires donc.
  • Pour moi intégrale double ou série de Fourier, c'est du même tonneau.
  • Salut, j'ai une question s'il vous plaît: pourquoi on n'a pas suivi le même ordre ici?
    Ps: c'est loin du sujet ici désolée mais je voyais inutile d'ouvrir une nouvelle discussion.79468
  • Un ensemble, ça n'a pas d'ordre : $\{1,2,3\}=\{2,3,1\}$, c'est l'ensemble qui contient $1$ et $2$ et $3$ et personne d'autre. Les propriétés de la somme (associativité et commutativité) font que justement, l'ordre des termes et l'ordre des opérations n'a pas d'importance : si on calcule $(a_1+a_2)+a_3$ ou $a_2+(a_3+a_1)$, on trouve le même résultat, noté indifféremment $\sum_{i\in\{1,2,3\}}a_i$ ou $\sum_{i\in\{2,3,1\}}a_i$ ou $a_1+a_3+a_2$.
  • Oui, mais dans la somme on voit apparaître des indices qui ne sont pas des éléments de I comme (1,1) et d'autres qui manquent comme (2,2).?
  • Alea a écrit:
    Pour moi intégrale double ou série de Fourier, c'est du même tonneau.

    Les goûts et les couleurs...

    Personnellement c'est ma période intégrale (comme pour d'autres c'est leur période bleue, rose...).
    Une belle solution est une solution qui n'utilise que des intégrales.
  • @FdP : comment obtiens-tu la somme de la série de terme général (-1)n/(2n+1)3 ? Merci.
  • @Totem:

    Il faut considérer l'intégrale

    $\begin{align} J_1=\int_0^\infty\frac{\ln^2 x}{1+x^2}\,dx&=\int_0^1\frac{\ln^2 x}{1+x^2},dx+\int_1^\infty\frac{\ln^2 x}{1+x^2}\,dx\\
    \end{align}$

    Dans la deuxième intégrale on fait le changement de variable $y=\dfrac{1}{x}$,

    $\begin{align} J_1=2\int_0^1\frac{\ln^2 x}{1+x^2}\,dx\end{align}$

    On développe en série $\dfrac{1}{1+x^2}$ et on intègre terme à terme et on obtient:

    $\displaystyle J_1=2\sum_{n=0}^\infty \frac{(-1)^n}{(2n+1)^3}$

    $J_1=J$ le $J$ est celui du message http://www.les-mathematiques.net/phorum/read.php?4,1698802,1698802#msg-1698802
  • OK. Il me semble qu'on peut retrouver cette somme avec les séries de Fourier mais je ne me souviens plus...

    C'est assez rare d'ailleurs et remarquable de trouver la valeur exacte d'une série quand l'exposant de n au dénominateur est impair !
  • Cette valeur était connue par Euler.
    Il l'a probablement "calculée" de la même façon qu'il a "calculé" pour la première fois la série des inverses des carrés des nombres entiers non nuls.
  • Et si on remplace l'exposant 3 par l'exposant 2 on tombe sur la constante de Catalan qui n' a par contre pas d'expression explicite !
  • Totem a écrit:
    on tombe sur la constante de Catalan qui n' a par contre pas d'expression explicite !

    Il faudrait que tu précises le sens de "expression explicite" pour pouvoir faire cette affirmation.

    Mais en effet, la méthode donnée ci-dessus ne fonctionne pas pour calculer:

    $\displaystyle \int_0^1 \frac{\ln^{2n+1} x}{1+x^2}\,dx$ avec $n\geq 0$ un entier.
  • Euh je voulais dire un résultat sans série ni intégrale, mais juste avec des nombres , les 4 opérations, des racines , et Pi et e bien sûr...!

    C'est vrai que c'est imprécis, est ce que sin ( Pi / 7) est une forme explicite ?
  • $\displaystyle \int_0^1 \dfrac{\arctan x}{x}\,dx$ est une expression explicite pour moi.
  • Tu as sans doute raison...même si on ne sait pas calculer de primitive ?
  • Une primitive de $\displaystyle \frac{\arctan x}{x}$ est $F(x)=\displaystyle \int_0^x \frac{\arctan t}{t}\,dt$
  • Un problème ouvert est si $\displaystyle \beta(2)=\sum_{n=0}^{\infty}\frac{(-1)^n}{(2n+1)^2}$ est irrationnel ou pas.

    L'un des résultats récents qui se rapproche d'une réponse positive:

    Parmi les nombres $\beta(2),\beta(4),\beta(6),\beta(8),\beta(10),\beta(12)$ l'un au moins est irrationnel.

    (cf. https://arxiv.org/pdf/1804.09922.pdf )

    Mais on est encore loin d'avoir une preuve que la constante de Catalan est un nombre irrationnel.
  • Fin de partie écrivait : http://www.les-mathematiques.net/phorum/read.php?4,1698802,1699984#msg-1699984
    [Inutile de recopier l'avant dernier message. Un lien suffit. AD]
    Bien sûr mais je voulais dire une primitive...explicite :-D.
    On ne dit pas forme close ?
  • Tiens, à propos, une intégrale que je trouve "mignonne":

    Soit $x$ un réel.
    Calculer $\quad\displaystyle \int_0^{\pi}\frac{\theta\sin \theta}{1+x^2\cos^2 \theta}\,d\theta$

    (Trouvée aujourd'hui dans un article de 1876)
  • Bonjour,

    L'intégrale $\displaystyle I=\int_{0}^{\pi } {\theta \sin \theta \over 1+x^2 \cos^2 \theta} d\theta $ existe pour toute valeur de $x$ réel. On change les variables $\displaystyle \theta \leadsto t$ avec $\displaystyle t=\pi-\theta$ et alors $\displaystyle I = \pi \int_{0}^{\pi} {\sin t \over 1+x^2 \cos^2 t} dt - I$ et alors $\displaystyle 2I = \pi \int_{0}^{\pi} {\sin t \over 1+x^2 \cos^2 t} dt =-\pi {\arctan (|x| \cos t) \over |x|}\mid_{0}^{\pi} = 2 \pi {\arctan x \over x}$ et donc $\displaystyle I = \pi {\arctan x \over x}$ avec prolongement par continuité en $\displaystyle x=0.$ Voilà !
  • Quelqu'un du forum m'a fait une blague? :-)

    ( https://math.stackexchange.com/questions/2893117/definite-integral-int-01-frac-ln4xx21-dx/2893947#2893947 )

    PS:

    Avec les notations ci-dessus on a pour $n\geq 0$,

    $\begin{align}\sum_{k=0}^{n}\binom{2n}{2k}J_kJ_{n-k}&=\left(2-\frac{1}{2^{2n+1}}\right)(2n+1)!\zeta(2n+2)\end{align}$

    En particulier la formule est vraie pour $n=0$ cela permet de retrouver la forme close standard pour $\zeta(2)$.
    ($J_0$ se calcule aisément)
  • On peut faire des trucs cool dans la même veine.

    Je me propose de "déguiser" l'intégrale $\displaystyle J=\int_0^{+\infty} \frac{1}{1+x^n}\,dx$, avec $n$ entier strictement plus grand que $1$, pour la rendre méconnaissable.

    On commence par effectuer le changement de variable $\displaystyle u=\dfrac{1}{x}$,

    $\begin{align}J&=\int_0^{\infty}\dfrac{u^{n-2}}{1+u^n}\,du\end{align}$


    $\begin{align}J^2&=\int_0^{\infty}\int_0^{\infty}\dfrac{(uy)^{n-2}}{(1+u^n)(1+y^n)}\,du\,dy\end{align}$

    On effectue le changement de variable $x=uy$ (la variable considérée est $u$),

    $\begin{align}J^2&=\int_0^{\infty}\int_0^{\infty}\dfrac{x^{n-2}y^{n-1}}{(y^n+u^n)(1+y^n)}\,dx\,dy\end{align}$

    On effectue le changement de variable $\displaystyle v=y^n$

    $\begin{align}J^2&=\frac{1}{n}\int_0^{\infty}\int_0^{\infty}\dfrac{x^{n-2}}{(v+u^n)(1+v)}\,dx\,dv\\
    &=\frac{1}{n}\int_0^{\infty}x^{n-2}\left[\frac{1}{x^n-1}\ln\left(\frac{v+1}{v+x^n}\right)\right]_{v=0}^{v=\infty}\,dx\\
    &=\int_0^{\infty}\frac{x^{n-2}\ln x}{x^n-1}\,dx\\
    &=\int_0^{1}\frac{x^{n-2}\ln x}{x^n-1}\,dx+\int_1^{\infty}\frac{x^{n-2}\ln x}{x^n-1}\,dx
    \end{align}$

    Dans la dernière intégrale on fait le changement de variable $z=\dfrac{1}{x}$,

    $\begin{align}\boxed{\displaystyle J^2=\int_0^{1}\frac{(z^{n-2}+1)\ln z}{z^n-1}\,dz}\end{align}$

    Cool non?
  • C'est très joli de relier $J$ et cette dernière intégrale.

    Une façon plus classique (mais plus longue) de le démontrer est d'introduire une série géométrique pour obtenir:
    $\displaystyle\int_0^{1}\frac{(z^{n-2}+1)\ln z}{z^n-1}\,dz=\sum_{k=0}^{+\infty}\left(\frac1{(nk+1)^2}+\frac1{(nk+n-1)^2}\right)=\frac1{n^2}(\Psi'(1/n)+\Psi'(1-1/n))$ où $\Psi(x)=\dfrac{\Gamma'(x)}{\Gamma(x)}$.
    Avec la relation $\Gamma(x)\Gamma(1-x)=\dfrac{\pi}{\sin (\pi x)}$ on déduit $\Psi'(x)+\Psi'(1-x)=\dfrac{\pi^2}{\sin ^2(\pi x)}$;

    Sachant que $J=\dfrac{\pi}{n\sin (\pi /n)}$ on obtient bien $J^2$.
  • En cherchant à calculer une intégrale j'ai établi d'une manière assez "simple", c'est à dire sans dilogarithme, sans intégrale double, sans dérivation sous le signe somme, la relation suivante qui ne me semble pas facile d'établir autrement:

    $\displaystyle \int_0^1 \frac{\ln(1-x)}{1+3x^2}\,dx=\frac{\pi\ln 2}{3\sqrt{3}}+\int_0^1 \frac{\ln x}{1+3x^2}\,dx$

    saurez-vous établir vous aussi simplement cette égalité?

    (je m'assure en posant la question qu'une méthode encore plus simple que celle à laquelle je pense a peu de chance d'exister)

    PS:
    La méthode utilisée permet sans doute d'établir un tas de formules du même type.

    PS2:
    Je ne sais calculer aucune des intégrales présentes dans cette égalité.

    PS3:

    Une autre formule encore plus bizarre établie par la même méthode:

    $\displaystyle \int_0^1 \dfrac{\ln(1-x)}{1+15x^2}\,dx=\dfrac{2\ln 2}{\sqrt{15}}\arctan\left(\sqrt{15}\right)+\int_0^1 \dfrac{\ln x}{1+15x^2}\,dx$

    Et bien sûr, je ne sais calculer aucune des deux intégrales de cette égalité.
  • L'explication (et justification) des deux égalités ci-dessous:

    Soit $a>1$,

    Soit $\displaystyle J(a)= \int_0^\infty \frac{\ln x}{\left(1+ax\right)^2+a^2-1}\,dx$

    On sait que $J(a)=0$

    (développer le dénominateur et faire le changement de variable $y=\frac{1}{x}$ )

    On effectue le changement de variable $y=\frac{1}{1+ax}$

    $\displaystyle J(a)= \int_0^1 \frac{\ln\left(\frac{1-y}{ay}\right)}{1+(a^2-1)y^2}\,dy$

    Ainsi,

    $\begin{align}\int_0^1 \frac{\ln\left(1-x\right)}{1+(a^2-1)y^2}\,dy&=\int_0^1 \frac{\ln y}{1+(a^2-1)y^2}\,dy+\ln a\int_0^1 \frac{1}{1+(a^2-1)y^2}\,dy\\
    &=\int_0^1 \frac{\ln y}{1+(a^2-1)y^2}\,dy+\ln a\frac{\arctan\left(\sqrt{a^2-1}\right)}{\sqrt{a^2-1}}
    \end{align}$
  • La technique décrite ci-dessus aide à démontrer le résultat suivant:

    $\displaystyle \int_0^{\frac{\pi}{6}} \ln^2\left(\frac{\sin x }{\sin\left(\frac{\pi}{6}-x\right)}\right)\,dx=\frac{35\pi^3}{648}$

    Saurez vous retrouver ce résultat?

    PS:
    Pourquoi ce résultat? C'est une étape vers une démonstration élémentaire de la formule:

    $\displaystyle \int_0^{\frac{\pi}{6}} \ln^2\left(2\sin x \right)\,dx=\frac{7\pi^3}{216}$

    (je ne sais pas si je vais aboutir)

    PS2:
    Les logarithmes sont tous élevés au carré.

    PS3:

    Il semble qu'on ait:

    $\displaystyle \int_0^{\frac{\pi}{6}} \ln^{2n}\left(\frac{\sin x }{\sin\left(\frac{\pi}{6}-x\right)}\right)\,dx=r_n\pi^{2n+1}$

    avec $r_n$ un rationnel.

    Je n'ai pas de preuve mais cela semble vrai pour $n=1,2,3,4,5$

    PS4:
    Je pense que la propriété énoncée au PS3 n'est pas plus difficile à démontrer que dans le cas $n=1$.

    PS5:
    Pour terminer la démonstration indiquée dans le PS1 il me reste à démontrer de façon élémentaire que:

    $\displaystyle \int_0^{\frac{\pi}{6}} \ln \left(2\sin x \right)\ln \left(2\sin\left(\frac{\pi}{6}-x \right)\right)\,dx=\frac{7\pi^3}{1296}$

    ou:

    $\displaystyle \int_0^{\frac{\pi}{6}} \ln^2 \left(4\sin x \sin\left(\frac{\pi}{6}-x \right)\right)\,dx=\frac{49\pi^3}{648}$

    Je suis assez confiant dans la faisabilité.
  • La propriété mentionnée ci-dessus semble encore plus générale.

    J'ai l'impression qu'on a:

    $\displaystyle \int_0^{a_n\pi} \ln^{2n}\left(\frac{\sin x }{\sin\left(a_n\pi-x\right)}\right)\,dx=r_n\pi^{2n+1}$

    Avec $n$ un entier naturel, $a_n$ un rationnel positif inférieur à $\frac{1}{2}$ et $r_n$ un rationnel.
  • Calcul de

    $\displaystyle J=\int_0^{\frac{\pi}{6}} \ln^2\left(\frac{\sin x }{\sin\left(\frac{\pi}{6}-x\right)}\right)\,dx$

    par des méthodes élémentaires (pas de séries de Fourier, pas de nombres complexes)

    On applique le changement de variable $y=\dfrac{\sin x }{\sin\left(\dfrac{\pi}{6}-x\right)}$,

    $\displaystyle J=\frac{1}{2}\int_0^{\infty} \frac{\ln^2 x}{x^2+\sqrt{3}x+1}\,dx$

    ($y$ peut s'exprimer en fonction de $\tan x$)

    Soit, pour $-2< \alpha<2$,

    $\displaystyle J(\alpha)=\frac{1}{2}\int_0^{\infty} \frac{\ln^2 x}{x^2+\alpha x+1}\,dx$

    On a $\displaystyle J=J(\sqrt{3})$

    $\displaystyle K(\alpha)=\int_0^{\infty} \frac{1}{x^2+\alpha x+1}\,dx$

    $\displaystyle L(\alpha)=\int_0^{\infty}\int_0^{\infty} \frac{\ln^2(xy)}{(x^2+\alpha x+1)(y^2+\alpha y+1)}\,dx\,dy$

    $\begin{align}K(\alpha)&=\left[\frac{2}{\sqrt{4-\alpha^2}}\arctan\left(\frac{2x+\alpha}{\sqrt{4-\alpha^2}}\right)\right]_0^\infty\\
    &=\frac{2}{\sqrt{4-\alpha^2}}\times \frac{\pi}{2}-\frac{2}{\sqrt{4-\alpha^2}}\times\arctan\left(\frac{\alpha}{\sqrt{4-\alpha^2}}\right)\\
    &=\frac{\pi}{\sqrt{4-\alpha^2}}-\frac{2}{\sqrt{4-\alpha^2}}\arctan\left(\frac{\alpha}{\sqrt{4-\alpha^2}}\right)\\
    \end{align}$

    $\begin{align}L(\alpha)&=\int_0^\infty\int_0^\infty \frac{\ln^2 x}{(x^2+\alpha x+1)(y^2+\alpha y+1)}\,dx\,dy+\int_0^\infty\int_0^\infty \frac{\ln^2 x}{(x^2+\alpha x+1)(y^2+\alpha y+1)}\,dx\,dy+\\
    &2\int_0^\infty\int_0^\infty \frac{\ln x\ln y}{(x^2+\alpha x+1)(y^2+\alpha y+1)}\,dx\,dy\\
    &=2\int_0^\infty\int_0^\infty \frac{\ln^2 x}{(x^2+\alpha x+1)(y^2+\alpha y+1)}\,dx\,dy\\
    &=4J(\alpha)K(\alpha)\\
    &=4\left(\frac{\pi}{\sqrt{4-\alpha^2}}-\frac{2}{\sqrt{4-\alpha^2}}\arctan\left(\frac{\alpha}{\sqrt{4-\alpha^2}}\right)\right)J(\alpha)\\
    &=\frac{4}{\sqrt{4-\alpha^2}}\left(\pi-2\arctan\left(\frac{\alpha}{\sqrt{4-\alpha^2}}\right)\right)J(\alpha)\\
    \end{align}$

    puisque $\displaystyle \int_0^\infty \frac{\ln x}{x^2+\alpha x+1}\,dx=0$ (faire le changement de variable $y=\dfrac{1}{x}$)

    Dans l'intégrale qui définit $L(\alpha)$ appliquer le changement de variable $u=xy$ (on s'intéresse à la variable $x$ ici),

    $\begin{align} L(\alpha)&=\int_0^\infty\int_0^\infty \frac{y\ln^2 u}{(y^2+\alpha y+1)(y^2+\alpha uy+u^2)}\,du\,dy\\
    &=\int_0^\infty \left[\frac{(u+1)\ln\left(\frac{y^2+\alpha y+1}{y^2+\alpha uy+u^2}\right)}{2(u^3+(1-\alpha^2)u^2-(1-\alpha^2)u-1)}-\frac{\alpha \left(\arctan\left(\frac{2y+\alpha u}{u\sqrt{4-\alpha^2}}\right)+\arctan\left(\frac{2y+\alpha}{\sqrt{4-\alpha^2}}\right)\right)}{(u^2+(2-\alpha^2)u+1)\sqrt{4-\alpha^2}}\right]_{y=0}^{y=\infty}\ln^2 u\,du\\
    &=\int_0^\infty \frac{\alpha\left(2\arctan\left(\frac{\alpha}{\sqrt{4-\alpha^2}}\right)-\pi\right)\ln^2 u}{\left(u^2+(2-a^2)u+1\right)\sqrt{4-\alpha^2}}\,du+\int_0^\infty \frac{(u+1)\ln^3 u}{u^3+(1-\alpha^2)u^2-(1-\alpha^2)u-1}\,du
    \end{align}$

    Donc,

    $\begin{align} J(\alpha)&=\frac{\sqrt{4-\alpha^2}}{4\left(\pi-2\arctan\left(\frac{\alpha}{\sqrt{4-\alpha^2}}\right)\right)}\int_0^\infty \frac{(u+1)\ln^3 u}{u^3+(1-\alpha^2)u^2-(1-\alpha^2)u-1}\,du-\frac{\alpha}{4}\int_0^\infty\frac{\ln^2 u}{u^2+(2-\alpha^2)u+1}\,du\\
    &=\frac{\sqrt{4-\alpha^2}}{4\left(\pi-2\arctan\left(\frac{\alpha}{\sqrt{4-\alpha^2}}\right)\right)}\int_0^\infty \frac{(u+1)\ln^3 u}{u^3+(1-\alpha^2)u^2-(1-\alpha^2)u-1}\,du-\frac{\alpha}{2}J(2-\alpha^2)\\
    \end{align}$

    A noter que,

    $\displaystyle J(1)=\dfrac{3\sqrt{3}}{8\pi}\int_0^\infty \dfrac{(x+1)\ln^3 x}{x^3-1}\,dx-\dfrac{1}{2}J(1)$

    Donc,

    $\displaystyle J(1)=\dfrac{\sqrt{3}}{4\pi}\int_0^\infty \dfrac{(x+1)\ln^3 x}{x^3-1}\,dx$

    Ainsi,

    $\begin{align} L(\sqrt{3})&=\int_0^\infty \frac{(u+1)\ln^3 u}{u^3-2u^2+2u-1}\,du-\frac{\pi}{\sqrt{3}}\int_0^\infty \frac{\ln^2 u}{u^2-u+1}\,du\\
    &=\int_0^\infty \frac{(u+1)\ln^3 u}{u^3-2u^2+2u-1}\,du-\frac{2\pi}{\sqrt{3}}J(-1)\\
    &=\int_0^\infty \frac{(u+1)\ln^3 u}{u^3-2u^2+2u-1}\,du-\frac{2\pi}{\sqrt{3}}\left(\frac{3\sqrt{3}}{16\pi}\int_0^\infty \frac{(u+1)\ln^3 u}{u^3-1}\,du+\frac{1}{2}J(1)\right)\\
    &=\int_0^\infty \frac{(u+1)\ln^3 u}{u^3-2u^2+2u-1}\,du-\frac{5}{8}\int_0^\infty \frac{(u+1)\ln^3 u}{u^3-1}\,du
    \end{align}$

    (A noter que les intégrandes sont invariantes par le changement de variable $y=\dfrac{1}{u}$)

    Sachant que $\displaystyle \dfrac{u+1}{u^2-2u^2+2u-1}=\dfrac{2u}{u^2-1}+\dfrac{1}{u-1}+\dfrac{3u^2}{u^3-1}-\dfrac{6u^5}{u^6-1}$

    et,

    $\displaystyle \dfrac{u+1}{u^3-1}=\dfrac{1}{u-1}-\dfrac{u^2}{u^3-1}$


    $\begin{align} L(\sqrt{3})&=\int_0^1 \frac{4u\ln^3 u}{u^2-1}\,du+\left(2-\frac{5}{4}\right)\int_0^1 \frac{\ln^3 u}{u-1}\,du+\left(6+\frac{5}{4}\right)\int_0^1 \frac{u^2 \ln^3 u}{u^3-1}\,du-\int_0^1 \frac{12u^5 \ln^3 u}{u^6-1}\,du\end{align}$

    Dans la première intégrale on fait le changement de variable $\displaystyle v=u^2$,
    Dans la troisième intégrale on fait le changement de variable $\displaystyle v=u^3$,
    Dans la quatrième intégrale on fait le changement de variable $\displaystyle v=u^6$,

    $\begin{align} L(\sqrt{3})&=\frac{2}{2^3}\int_0^1 \frac{\ln^3 u}{u-1}\,du+\frac{3}{4}\int_0^1 \frac{\ln^3 u}{u-1}\,du+\frac{29}{4\times 3\times 3^3}\int_0^1 \frac{\ln^3 u}{u-1}\,du-\frac{2}{6^3}\int_0^1 \frac{\ln^3 u}{u-1}\,du\\
    &=\frac{175}{162}\int_0^1 \frac{\ln^3 u}{u-1}\,du\\
    \end{align}$

    or,

    $\displaystyle \int_0^1 \frac{\ln^3 u}{u-1}\,du=6\zeta(4)$ (développement en série)

    donc,

    $\begin{align} L(\sqrt{3})&=\frac{175}{27}\zeta(4)\end{align}$

    Ainsi,

    $\begin{align} J(\sqrt{3})&=\frac{1}{\frac{4\pi}{3}}\times \frac{175}{27}\zeta(4)\\
    &=\frac{175}{36\pi}\zeta(4)\\
    \end{align}$

    Si on sait que $\displaystyle \zeta(4)=\dfrac{\pi^4}{90}$

    $\begin{align} J(\sqrt{3})&=\frac{175}{36\pi}\times \dfrac{\pi^4}{90}\\
    &=\boxed{\dfrac{35\pi^3}{648}}
    \end{align}$


    PS:
    Je ne sais pas (encore) calculer $\displaystyle \int_0^{\frac{\pi}{6}} \ln\left(2\sin x\right)\ln\left(2\sin\left(\frac{\pi}{6}-x\right)\right)\,dx$ ou $\displaystyle \int_0^{\frac{\pi}{6}} \ln\left(4\sin x\sin\left(\frac{\pi}{6}-x\right)\right)^2\,dx$ de façon élémentaire.

    PS2:
    En fait, dans le cours du calcul on peut voir esquissée une démonstration (élémentaire) que: $\displaystyle \int_0^\infty \frac{\ln^2 x}{x^2-x+1}\,dx=\dfrac{20\pi^3}{81\sqrt{3}}$ et il y a le calcul de $\displaystyle \int_0^\infty \frac{\ln^2 x}{x^2+x+1}\,dx$
Connectez-vous ou Inscrivez-vous pour répondre.